K
Khách

Hãy nhập câu hỏi của bạn vào đây, nếu là tài khoản VIP, bạn sẽ được ưu tiên trả lời.

5 tháng 1 2018

Ta có: \(\left\{{}\begin{matrix}0< 24< 25\\0< 35< 36\end{matrix}\right.\)

\(\Rightarrow\left\{{}\begin{matrix}\sqrt{24}< \sqrt{25}\\\sqrt{35}< \sqrt{36}\end{matrix}\right.\)

\(\Leftrightarrow\left\{{}\begin{matrix}\sqrt{24}< 5\\\sqrt{35}< 6\end{matrix}\right.\)

\(\Rightarrow\sqrt{24}+\sqrt{35}\) < 5 + 6

\(\Leftrightarrow\) \(\sqrt{24}+\sqrt{35}\) < 11

Vậy \(\sqrt{24}+\sqrt{35}\) < 11

7 tháng 1 2018

mi làm sai rồi

7 tháng 1 2018

óc chó\

28 tháng 7 2016

a) Ta có

\(\sqrt{35}< \sqrt{36}=6\)

\(\sqrt{99}< \sqrt{100}=10\)

\(\Rightarrow\sqrt{35}+\sqrt{99}< 10+6=16\)

b) Ta có

\(\sqrt{50}>\sqrt{49}=7\)

\(\sqrt{17}>\sqrt{16}=4\)

\(\Rightarrow\sqrt{50}+\sqrt{17}>7+4=11\)

28 tháng 7 2016

là vậy à ,cảm ơn nhen

28 tháng 7 2016

bài này mk làm bên dưới rồi 

bạn kéo xuống là thấy nhé

28 tháng 7 2016

bạn cũng có thể ấn vào Câu hỏi của CON CHÓ 4 ĐẦU - Toán lớp 7 | Học trực tuyến

\(\sqrt{24}+\sqrt{35}+\sqrt{99}

14 tháng 10 2021

\(\left\{{}\begin{matrix}a=\dfrac{35}{49}=\dfrac{5}{7}\\b=\sqrt{\dfrac{5^2}{7^2}}=\dfrac{5}{7}\\c=\dfrac{\sqrt{5^2}+\sqrt{35^2}}{\sqrt{7^2}+\sqrt{49^2}}=\dfrac{5+35}{7+49}=\dfrac{5}{7}\\d=\dfrac{\sqrt{5^2}-\sqrt{35^2}}{\sqrt{7^2}-\sqrt{49^2}}=\dfrac{5-35}{7-49}=\dfrac{5}{7}\end{matrix}\right.\)

\(\Rightarrow a=b=c=d=\dfrac{5}{7}\)

14 tháng 10 2021

\(a=\dfrac{35}{49};b=\dfrac{5}{7}\\ c,=\dfrac{5+35}{7+49}=\dfrac{12}{14}=\dfrac{6}{7}\\ d,=\dfrac{5-35}{7-49}\)

Áp dụng t/c dtsbn:

\(\dfrac{5}{7}=\dfrac{35}{49}=\dfrac{5+35}{7+49}=\dfrac{5-35}{7-49}\) hay \(a=b=c=d\)

 

14 tháng 10 2018

Ta có : \(\sqrt{61-35}=\sqrt{26}>\sqrt{25}=5\)(1)

           \(\sqrt{61}-\sqrt{35}< \sqrt{64}-\sqrt{36}=8-6=2\)(2)

Từ (1) và (2) ta được :  \(\sqrt{61-35}>5>2>\sqrt{61}-\sqrt{35}\)

\(\Rightarrow\sqrt{61-35}>\sqrt{61}-\sqrt{35}\)

31 tháng 10 2018

ghi de sai ban oi

31 tháng 10 2018

\(A=\sqrt{2}+\sqrt{6}+\sqrt{12}+\sqrt{20}+\sqrt{30}+\sqrt{42}\)

\(A< \sqrt{2,25}+\sqrt{6,25}+\sqrt{12,25}+\sqrt{20,25}+\sqrt{30,25}+\sqrt{42,25}=24=B\)

Vậy \(A< B\)

Chúc bạn học tốt ~